Skip to main content
Mathematics LibreTexts

2.2: Limit Theorems

  • Page ID
    49100
  • \( \newcommand{\vecs}[1]{\overset { \scriptstyle \rightharpoonup} {\mathbf{#1}} } \) \( \newcommand{\vecd}[1]{\overset{-\!-\!\rightharpoonup}{\vphantom{a}\smash {#1}}} \)\(\newcommand{\id}{\mathrm{id}}\) \( \newcommand{\Span}{\mathrm{span}}\) \( \newcommand{\kernel}{\mathrm{null}\,}\) \( \newcommand{\range}{\mathrm{range}\,}\) \( \newcommand{\RealPart}{\mathrm{Re}}\) \( \newcommand{\ImaginaryPart}{\mathrm{Im}}\) \( \newcommand{\Argument}{\mathrm{Arg}}\) \( \newcommand{\norm}[1]{\| #1 \|}\) \( \newcommand{\inner}[2]{\langle #1, #2 \rangle}\) \( \newcommand{\Span}{\mathrm{span}}\) \(\newcommand{\id}{\mathrm{id}}\) \( \newcommand{\Span}{\mathrm{span}}\) \( \newcommand{\kernel}{\mathrm{null}\,}\) \( \newcommand{\range}{\mathrm{range}\,}\) \( \newcommand{\RealPart}{\mathrm{Re}}\) \( \newcommand{\ImaginaryPart}{\mathrm{Im}}\) \( \newcommand{\Argument}{\mathrm{Arg}}\) \( \newcommand{\norm}[1]{\| #1 \|}\) \( \newcommand{\inner}[2]{\langle #1, #2 \rangle}\) \( \newcommand{\Span}{\mathrm{span}}\)\(\newcommand{\AA}{\unicode[.8,0]{x212B}}\)

    We now prove several theorems that facilitate the computation of limits of some sequences in terms of those of other simpler sequences.

    Theorem \(\PageIndex{1}\)

    Let \(\left\{a_{n}\right\}\) and \(\left\{b_{n}\right\}\) be sequences of real numbesr and let \(k\) be a real number. Suppose \(\left\{a_{n}\right\}\) converges to \(a\) and \(\left\{b_{n}\right\}\) converges to \(b\). Then the sequences \(\left\{a_{n}+b_{n}\right\}\), \(\left\{k a_{n}\right\}\), and \(\left\{a_{n} b_{n}\right\}\) converge and

    1. \(\lim _{n \rightarrow \infty}\left(a_{n}+b_{n}\right)=a+b\);
    2. \(\lim _{n \rightarrow \infty}\left(k a_{n}\right)=k a\);
    3. \(\lim _{n \rightarrow \infty}\left(a_{n} b_{n}\right)=a b\);
    4. If in addition \(b \neq 0\) and \(b_{n} \neq 0\) for \(n \in \mathbb{N}\)
    Proof

    (a) Fix any \(\varepsilon>0\). Since \(\left\{a_{n}\right\}\) converges to \(a\), there exists \(N_{1} \in \mathbb{N}\) such that

    \[\left|a_{n}-a\right|<\frac{\varepsilon}{2} \text { for all } n \geq N_{1}.\]

    Similarly, there exists \(N_{2} \in \mathbb{N}\) such that

    \[\left|b_{n}-b\right|<\frac{\varepsilon}{2} \text { for all } n \geq N_{2}.\]

    Let \(N=\max \left\{N_{1}, N_{2}\right\}\). For any \(n \geq N\), one has

    \[\left|\left(a_{n}+b_{n}\right)-(a+b)\right| \leq\left|a_{n}-a\right|+\left|b_{n}-b\right|<\frac{\varepsilon}{2}+\frac{\varepsilon}{2}=\varepsilon.\]

    Therefore, \(\lim _{n \rightarrow \infty}\left(a_{n}+b_{n}\right)=a+b\). This proves (a).

    (b) If \(k=0\), then \(ka=0\) and \(k a_{n}=0\) for all \(n\). The conclusion follows immediately. Suppose next that \(k \neq 0\). Given \(\varepsilon>0\), let \(N \in \mathbb{N}\) be such that \(\left|a_{n}-a\right|<\frac{\varepsilon}{|k|}\) for \(n \geq N\). Then for \(n \geq N\), \(\left|k a_{n}-k a\right|=|k|\left|a_{n}-a\right|<\varepsilon\). It follows that \(\lim _{n \rightarrow \infty}\left(k a_{n}\right)=k a\) as desired. This proves (b).

    (c) Since \(\left\{a_{n}\right\}\) is convergent, it follows from Theorem 2.1.7 that is bounded. Thus, there exists \(M>0\) such that

    \[\left|a_{n}\right| \leq M \text { for all } n \in \mathbb{N}.\]

    For every \(n \in \mathbb{N}\), we have the following estimate:

    \[\left|a_{n} b_{n}-a b\right|=\left|a_{n} b_{n}-a_{n} b+a_{n} b-a b\right| \leq\left|a_{n}\right|\left|b_{n}-b\right|+|b|\left|a_{n}-a\right|.\]

    Let \(\varepsilon>0\). Since \(\left\{a_{n}\right\}\) converges to \(a\), we may choose \(N_{1} \in \mathbb{N}\) such that

    \[\left|a_{n}-a\right|<\frac{\varepsilon}{2(|b|+1)} \text { for all } n \geq N_{1}.\]

    Similarly, since \(\left\{b_{n}\right\}\) converges to \(b\), we may choose \(N_{2} \in \mathbb{N}\) such that

    \[\left|b_{n}-b\right|<\frac{\varepsilon}{2 M} \text { for all } n \geq N_{2}.\]

    Let \(N=\max \left\{N_{1}, N_{2}\right\}\). Then, for \(n \geq N\), it follows from (2.1) that

    \[\left|a_{n} b_{n}-a b\right|<M \frac{\varepsilon}{2 M}+|b| \frac{\varepsilon}{2(|b|+1)}<\varepsilon \text { for all } n \geq N.\]

    Therefore, \(\lim _{n \rightarrow \infty} a_{n} b_{n}=a b\). This proves (c).

    (d) Let us first show that

    \[\lim _{n \rightarrow \infty} \frac{1}{b_{n}}=\frac{1}{b}.\]

    Since \(\left\{b_{n}\right\}\) converges to \(b\), there is \(N_{1} \in \mathbb{N}\) that

    \[\left|b_{n}-b\right|<\frac{|b|}{2} \text { for } n \geq N_{1}.\]

    It follows (using a triangle inequality) that, for such \(n\), \(-\frac{|b|}{2}<\left|b_{n}\right|-|b|<\frac{|b|}{2}\) and, hence, \(\frac{|b|}{2}<\left|b_{n}\right|\). For each \(n \geq N_{1}\), we have the following estimate

    \[\left|\frac{1}{b_{n}}-\frac{1}{b}\right|=\frac{\left|b_{n}-b\right|}{\left|b_{n}\right||b|} \leq \frac{2\left|b_{n}-b\right|}{b^{2}}.\]

    Now let \(\varepsilon>0\). Since \(\lim _{n \rightarrow \infty} b_{n}=b\), there exists \(N_{2} \in \mathbb{N}\) such that

    \[\left|b_{n}-b\right|<\frac{b^{2} \varepsilon}{2} \text { for all } n \geq N_{2}.\]

    Let \(N=\max \left\{N_{1}, N_{2}\right\}\). By (2.2), one has

    \[\left|\frac{1}{b_{n}}-\frac{1}{b}\right| \leq \frac{2\left|b_{n}-b\right|}{b^{2}}<\varepsilon \text { for all } n \geq N.\]

    It follows that \(\lim _{n \rightarrow \infty} \frac{1}{b_{n}}=\frac{1}{b}\).

    Finally, we can apply part (c) and have

    \[\lim _{n \rightarrow \infty} \frac{a_{n}}{b_{n}}=\lim _{n \rightarrow \infty} a_{n} \frac{1}{b_{n}}=\frac{a}{b}.\]

    The proof is now complete. \(\square\)

    Example \(\PageIndex{1}\)

    Consider the sequence \(\left\{a_{n}\right\}\) given by

    \[a_{n}=\frac{3 n^{2}-2 n+5}{1-4 n+7 n^{2}}.\]

    Solution

    Dividing numerator and denominator by \(n^{2}\), we can write

    \[a_{n}=\frac{3-2 / n+5 / n^{2}}{1 / n^{2}-4 / n+7}\]

    Therefore, by the limit theorems above,

    \[\lim _{n \rightarrow \infty} a_{n}=\lim _{n \rightarrow \infty} \frac{3-2 / n+5 / n^{2}}{1 / n^{2}-4 / n+7}=\frac{\lim _{n \rightarrow \infty} 3-\lim _{n \rightarrow \infty} 2 / n+\lim _{n \rightarrow \infty} 5 / n^{2}}{\lim _{n \rightarrow \infty} 1 / n^{2}-\lim _{n \rightarrow \infty} 4 / n+\lim _{n \rightarrow \infty} 7}=\frac{3}{7}.\]

    Example \(\PageIndex{2}\)

    Let \(a_{n}=\sqrt[n]{b}\), where \(b>0\). Consider the case where \(b>1\).

    Solution

    In this case, \(a_{n}>1\) for every \(n\). By the binomial theorem,

    \[b=a_{n}^{n}=\left(a_{n}-1+1\right)^{n} \geq 1+n\left(a_{n}-1\right).\]

    This implies

    \[0<a_{n}-1 \leq \frac{b-1}{n}.\]

    For each \(\varepsilon>0\), choose \(N>\frac{b-1}{\varepsilon}\). It follows that for \(n \geq N\),

    \[\left|a_{n}-1\right|=a_{n}-1<\frac{b-1}{n} \leq \frac{b-1}{N}<\varepsilon.\]

    Thus, \(\lim _{n \rightarrow \infty} a_{n}=1\).

    In the case where \(b=1\), it is obvious that \(a_{n}=1\) for all \(n\) and, hence, \(\lim _{n \rightarrow \infty} a_{n}=1\).

    If \(0<b<1\), let \(c=\frac{1}{b}\) and define

    \[x_{n}=\sqrt[n]{c}=\frac{1}{a_{n}}.\]

    Since \(c>1\), it has been shown that \(\lim _{n \rightarrow \infty} x_{n}=1\). This implies

    \[\lim _{n \rightarrow \infty} a_{n}=\lim _{n \rightarrow \infty} \frac{1}{x_{n}}=1.\]

    Exercise \(\PageIndex{1}\)

    Find the following limits:

    1. \(\lim _{n \rightarrow \infty} \frac{3 n^{2}-6 n+7}{4 n^{2}-3}\),
    2. \(\lim _{n \rightarrow \infty} \frac{1+3 n-n^{3}}{3 n^{3}-2 n^{2}+1}\).
    Answer

    Add texts here. Do not delete this text first.

    Exercise \(\PageIndex{2}\)

    Find the following limts:

    1. \(\lim _{n \rightarrow \infty} \frac{\sqrt{3 n}+1}{\sqrt{n}+\sqrt{3}}\),
    2. \(\lim _{n \rightarrow \infty} \sqrt[n]{\frac{2 n+1}{n}}\).
    Answer

    Add texts here. Do not delete this text first.

    Exercise \(\PageIndex{3}\)

    Find the following limits if they exist:

    1. \(\lim _{n \rightarrow \infty}\left(\sqrt{n^{2}+n}-n\right)\).
    2. \(\lim _{n \rightarrow \infty}\left(\sqrt[3]{n^{3}+3 n^{2}}-n\right)\).
    3. \(\lim _{n \rightarrow \infty}\left(\sqrt[3]{n^{3}+3 n^{2}}-\sqrt{n^{2}+n}\right)\)
    4. \(\lim _{n \rightarrow \infty}(\sqrt{n+1}-\sqrt{n})\).
    5. \(\lim _{n \rightarrow \infty}(\sqrt{n+1}-\sqrt{n}) / n\).
    Answer

    Add texts here. Do not delete this text first.

    Exercise \(\PageIndex{4}\)

    Find the following limts.

    1. For \(|r|<1\) and \(b \i \mathbb{R}\), \(\lim _{n \rightarrow \infty}\left(b+b r+b r^{2}+\cdots+b r^{n}\right)\).
    2. \(\lim _{n \rightarrow \infty}\left(\frac{2}{10}+\frac{2}{10^{2}}+\cdots+\frac{2}{10^{n}}\right)\).
    Answer

    Add texts here. Do not delete this text first.

    Exercise \(\PageIndex{5}\)

    Prove or disprove the following statements:

    1. If \(\left\{a_{n}\right\}\) and \(\left\{b_{n}\right\}\) are convergent sequences, then \(\left\{a_{n}+b_{n}\right\}\) is a convergent sequence.
    2. If \(\left\{a_{n}\right\}\) and \(\left\{b_{n}\right\}\) are divergent sequences, then \(\left\{a_{n}+b_{n}\right\}\) is a divergent sequence.
    3. If \(\left\{a_{n}\right\}\) and \(\left\{b_{n}\right\}\) are convergent sequences, then \(\left\{a_{n} b_{n}\right\}\) is a convergent sequence.
    4. If \(\left\{a_{n}\right\}\) and \(\left\{b_{n}\right\}\) are divergent sequences, then \(\left\{a_{n} b_{n}\right\}\) is a divergent sequence.
    5. If \(\left\{a_{n}\right\}\) and \(\left\{a_{n}+b_{n}\right\}\) are convergent sequences, then \(\left\{b_{n}\right\}\) is a convergent sequence.
    6. If \(\left\{a_{n}\right\}\) and \(\left\{a_{n}+b_{n}\right\}\) are divergent sequences, then \(\left\{b_{n}\right\}\) is a divergent sequence.
    Answer

    Add texts here. Do not delete this text first.


    This page titled 2.2: Limit Theorems is shared under a CC BY-NC-SA license and was authored, remixed, and/or curated by Lafferriere, Lafferriere, and Nguyen (PDXOpen: Open Educational Resources) .

    • Was this article helpful?